Hermitian Operators: Referencing Griffiths

In summary: If you are working with a matrix, then complex conjugation is one way of taking the adjoint. If you are working with operators on functions, then the adjoint is taken by integrating over the appropriate inner product, which is usually complex. In any case, you need to be certain that the adjoint of the operator is equal to the complex conjugate of the operator, and that depends on the operator.So, in summary, if you are trying to show that an operator is Hermitian, then you usually need to show that the operator is self-adjoint. That means you need to show that ##\hat{Q} = \hat{Q}^\dagger##. How you
  • #1
WWCY
479
12
I have a few issues with understanding a section of Griffiths QM regarding Hermitian Operators and would greatly appreciate some help.

It was first stated that,

##\langle Q \rangle = \int \Psi ^* \hat{Q} \Psi dx = \langle \Psi | \hat{Q} \Psi \rangle##

and because expectation values are real,

##\langle Q \rangle = \langle Q \rangle ^*##

By invoking ##\langle f|g \rangle = \langle g|f \rangle ^*## (1)

##\langle \Psi | \hat{Q} \Psi \rangle = \langle \Psi | \hat{Q} \Psi \rangle ^* = \langle \hat{Q} \Psi| \Psi \rangle ##.

Which I so far, seem to understand. But then it was stated that,

##\langle f| \hat{Q}g \rangle = \langle \hat{Q}f|g \rangle## was the condition for a Hermitian operator.

Up till now my understanding (which seems plainly wrong) was as follows,

##\langle f | \hat{Q} g\rangle = \langle f | \hat{Q} g\rangle ^* = \langle \hat{Q}g|f \rangle ##

where step 2 to 3 involved the flipping of functions as seen in (1).

Could anyone explain how it's supposed to work? Assistance is greatly appreciated!

P.S. It would be nice if explanations could be kept simple, I have not worked up till all things "eigen" (next section of book) as of yet. Thanks!
 
Physics news on Phys.org
  • #2
This is always true ##\langle f|g \rangle = \langle g|f \rangle ^*##.
Now consider ##\langle \Phi | \hat{Q} \Psi \rangle##. Define ## | \hat{Q} \Psi \rangle = | \Omega \rangle##. Then
##\langle \Phi |\Omega\rangle = \langle \Omega| \Phi \rangle ^*## is always true which means that ##\langle \Phi |\hat{Q}\Psi\rangle = \langle \hat{Q} \Psi| \Phi \rangle ^*## is also always true.

However, if it is also true that ##\langle \Phi | \hat{Q}\Psi\rangle = \langle \hat{Q} \Psi| \Phi \rangle ##, then ##\hat{Q}## is hermitian. The converse is also true. If ##\hat{Q}## is hermitian, then ##\langle \Phi | \hat{Q}\Psi\rangle = \langle \hat{Q} \Psi| \Phi \rangle ##.

That's how it works.
 
  • Like
Likes Nugatory
  • #3
kuruman said:
This is always true ##\langle f|g \rangle = \langle g|f \rangle ^*##.
Now consider ##\langle \Phi | \hat{Q} \Psi \rangle##. Define ## | \hat{Q} \Psi \rangle = | \Omega \rangle##. Then
##\langle \Phi |\Omega\rangle = \langle \Omega| \Phi \rangle ^*## is always true which means that ##\langle \Phi |\hat{Q}\Psi\rangle = \langle \hat{Q} \Psi| \Phi \rangle ^*## is also always true.

However, if it is also true that ##\langle \Phi | \hat{Q}\Psi\rangle = \langle \hat{Q} \Psi| \Phi \rangle ##, then ##\hat{Q}## is hermitian. The converse is also true. If ##\hat{Q}## is hermitian, then ##\langle \Phi | \hat{Q}\Psi\rangle = \langle \hat{Q} \Psi| \Phi \rangle ##.

That's how it works.

Thanks @kuruman , that helped clear things up a bit. However I still don't get the flow of logic behind this##\langle f| \hat{Q}g \rangle = \langle \hat{Q}f|g \rangle ##. What allows us to swap only the operator?
 
  • #4
I don't like that "operator in ket" notation, but let's simply take it as a shorthand for ##|\hat{Q}g \rangle = \hat{Q} |g \rangle##. Then, for ##\hat{Q} = \hat{Q}^\dagger##
$$
\begin{align*}
\langle f | \hat{Q}g \rangle &= \langle f | \hat{Q} |g \rangle \\
&= \left[\langle f | \hat{Q} |g \rangle^\dagger \right]^\dagger \\
&= \left[\langle g | \hat{Q}^\dagger |f \rangle \right]^\dagger \\
&= \left[\langle g | \hat{Q} |f \rangle \right]^\dagger \\
&= \left[\langle g | \hat{Q} f \rangle \right]^\dagger \\
&= \langle \hat{Q} f | g \rangle
\end{align*}
$$
 
  • Like
Likes WWCY
  • #5
DrClaude said:
I don't like that "operator in ket" notation, but let's simply take it as a shorthand for ##|\hat{Q}g \rangle = \hat{Q} |g \rangle##. Then, for ##\hat{Q} = \hat{Q}^\dagger##
$$
\begin{align*}
\langle f | \hat{Q}g \rangle &= \langle f | \hat{Q} |g \rangle \\
&= \left[\langle f | \hat{Q} |g \rangle^\dagger \right]^\dagger \\
&= \left[\langle g | \hat{Q}^\dagger |f \rangle \right]^\dagger \\
&= \left[\langle g | \hat{Q} |f \rangle \right]^\dagger \\
&= \left[\langle g | \hat{Q} f \rangle \right]^\dagger \\
&= \langle \hat{Q} f | g \rangle
\end{align*}
$$

So based on post 2, ##\langle \hat{Q} f | g \rangle = langle \hat{Q} g | f \rangle##?

Also, why does ##\hat{Q} = \hat{Q}^\dagger##? If we were talking about the momentum operator, does it not change its sign due to the ##i## it has?

Thank you for your assistance and patience.
 
  • #6
WWCY said:
So based on post 2, ##\langle \hat{Q} f | g \rangle = \langle \hat{Q} g | f \rangle##?
I would say no, it's ##\langle \hat{Q} f | g \rangle = \langle \hat{Q} g | f \rangle^\dagger##. What @kuruman wrote is not obvious to me; maybe he can clarify.

WWCY said:
Also, why does ##\hat{Q} = \hat{Q}^\dagger##?
Because we are discussing Hermitian operators:
WWCY said:
I have a few issues with understanding a section of Griffiths QM regarding Hermitian Operators

WWCY said:
If we were talking about the momentum operator, does it not change its sign due to the ##i## it has?
See https://www.physicsforums.com/threads/hermitian-momentum-operator.742281/
 
  • #7
I too dislike "operators in kets" notation, but I stuck with it to reply consistently with OP's initial question. I tried to sort this out thinking of the brakets as integrals. In terms of integrals, an operator ##\hat{Q}## is hermitian iff
$$\int{\Phi^*(\hat{Q}\Psi)~dx}=\int{(\hat{Q}\Phi)^*\Psi~dx}$$
which in post #2 I incorrectly translated to "operators in kets" notation. I should have written
$$\langle \Phi | \hat{Q}\Psi\rangle = \langle \hat{Q} \Phi| \Psi \rangle$$
Sorry about the confusion I may have caused.
 
  • Like
Likes DrClaude
  • #8
Thanks for the clarification!

However @DrClaude , I couldn't follow the arguments in the thread as it was still too technical for me.

Can I take it that ##\hat{Q} = \hat{Q}^*## is a condition for a hermitian operator?
 
  • #9
WWCY said:
Can I take it that ##\hat{Q} = \hat{Q}^*## is a condition for a hermitian operator?
Generally speaking, a Hermitian operator follows ##\hat{Q} = \hat{Q}^\dagger##. What the other thread pointed out is that you have to be careful what the ##\dagger## (Hermitian conjugation) does, as in some cases it is not simply equal to complex conjugation (as for the momentum operator).
 
  • Like
Likes Leo1233783
  • #10
Let me attempt to clarify with a couple of concrete examples. Suppose you were asked to show that an operator is Hermitian. What do you need to do? The answer is "it depends on the operator." If the operator can be represented by a matrix, such as spin, then you need to show that the matrix representing the operator is self-adjoint. OK what does that mean? It means you need to show that ##\hat{Q} = \hat{Q}^\dagger##. OK and what does that mean. It means that if you interchange rows and columns of the matrix and at the same time you take the complex conjugate of the elements, you end up with the same matrix.
Example 1. Consider two matrices
$$M_1=\begin{pmatrix}
0 &i \\
i & 0
\end{pmatrix}
~~~M_2=\begin{pmatrix}
0 &-i \\
i & 0
\end{pmatrix}
$$Note that
##M_1^{\dagger}=\begin{pmatrix}
0 &-i \\
-i & 0
\end{pmatrix}##, from which ##M_1\neq M_1^{\dagger}##, therefore ##M_1## is not Hermitian. If you apply the same test to ##M_2##, you will see that it is indeed Hermitian.
Example 2. To show that an operator such as momentum is Hermitian, you cannot use the matrix representation. You need to show that
$$\int_{- \infty}^{\infty}{\Phi^*(x) \left( \frac{\hbar}{i} \frac{\partial \Psi(x)}{\partial x} \right) ~dx}=\int_{- \infty}^{\infty}{\left( \frac{\hbar}{i} \frac{\partial \Phi(x)}{\partial x} \right)^*\Psi(x)~dx}$$
This can be done using integration by parts (twice) and assuming that ##\Phi(x)## and ##\Psi(x)## are normalizable and don't do anything horrible as ##x \rightarrow \pm \infty##. Operator ##\hat{x}## is clearly Hermitian because it's real.

There is more that can be said about all this, but I will stop here. It should become clearer to you as you delve deeper into QM and develop some familiarity.

On Edit: When I taught QM out of Griffiths, I introduced Hermitian operators and their properties using integrals and before introducing Dirac notation. This seemed to be less confusing to students.
 
  • Like
Likes DrClaude and WWCY
  • #11
DrClaude said:
Generally speaking, a Hermitian operator follows ##\hat{Q} = \hat{Q}^\dagger##. What the other thread pointed out is that you have to be careful what the ##\dagger## (Hermitian conjugation) does, as in some cases it is not simply equal to complex conjugation (as for the momentum operator).

kuruman said:
Example 2. To show that an operator such as momentum is Hermitian, you cannot use the matrix representation. You need to show that
$$\int_{- \infty}^{\infty}{\Phi^*(x) \left( \frac{\hbar}{i} \frac{\partial \Psi(x)}{\partial x} \right) ~dx}=\int_{- \infty}^{\infty}{\left( \frac{\hbar}{i} \frac{\partial \Phi(x)}{\partial x} \right)^*\Psi(x)~dx}$$
This can be done using integration by parts (twice) and assuming that ##\Phi(x)## and ##\Psi(x)## are normalizable and don't do anything horrible as ##x \rightarrow \pm \infty##. Operator ##\hat{x}## is clearly Hermitian because it's real.

Thank you both for your responses.

Would I then be right in saying that I should test operators by working them out in integral form (for now)?
 
  • #12
WWCY said:
Would I then be right in saying that I should test operators by working them out in integral form (for now)?
For now yes, until you go deep in chapter 3 of Griffiths where he starts using Dirac notation.
 
  • #13
Again a warning: It's not sufficient for an operator representing an observable to be Hermitean. It must be even self-adjoint. For details, see the Marvelous pedagogical papers

F. Gieres, Mathematical surprises and Dirac's formalism in quantum mechanics, Rep. Prog. Phys., 63 (2000), p. 1893.
http://arxiv.org/abs/quant-ph/9907069

G. Bonneau and J. Faraut, Self-adjoint extensions of operators and the teaching of quantum mechanics, Am. Jour. Phys., 69 (2001), p. 322.
http://arxiv.org/abs/quant-ph/0103153

It seems to me, Griffiths's QT book is sometimes a bit (too?) imprecise, but I've not yet have had a close look at it. So maybe that's an unjustified impression from many discussions related to this textbook in the forums.
 
  • Like
Likes DrClaude
  • #14
vanhees71 said:
Again a warning: It's not sufficient for an operator representing an observable to be Hermitean. It must be even self-adjoint. For details, see the Marvelous pedagogical papers

F. Gieres, Mathematical surprises and Dirac's formalism in quantum mechanics, Rep. Prog. Phys., 63 (2000), p. 1893.
http://arxiv.org/abs/quant-ph/9907069

G. Bonneau and J. Faraut, Self-adjoint extensions of operators and the teaching of quantum mechanics, Am. Jour. Phys., 69 (2001), p. 322.
http://arxiv.org/abs/quant-ph/0103153

It seems to me, Griffiths's QT book is sometimes a bit (too?) imprecise, but I've not yet have had a close look at it. So maybe that's an unjustified impression from many discussions related to this textbook in the forums.

Thank you, I'll keep these in the back of my head till I'm able to go through them.

Griffith's book does seem that way to me, but I don't have an alternative that is targetted at novices like myself. Do you have any recommendations?
 
  • #15
My favorite for introductory non-relativistic QM is J. J. Sakurai, Modern Quantum Mechanics (2nd edition).
 

1. What is a Hermitian operator?

A Hermitian operator is a type of linear operator used in quantum mechanics. It is defined as an operator that is equal to its own adjoint (conjugate transpose). In other words, the operator and its adjoint have the same matrix elements.

2. What is the significance of Hermitian operators in quantum mechanics?

Hermitian operators have several important properties that make them useful in quantum mechanics. They are associated with observables, meaning that their expectation values correspond to measurable quantities in physical systems. They also have real eigenvalues, which are important in solving the Schrödinger equation.

3. How are Hermitian operators related to unitary operators?

A unitary operator is a type of linear operator that preserves the inner product of vectors. All unitary operators are also Hermitian, but not all Hermitian operators are unitary. Unitary operators play an important role in quantum mechanics, as they represent transformations that conserve probabilities.

4. Can Hermitian operators have complex eigenvalues?

No, Hermitian operators must have real eigenvalues. This is because the eigenvalues of a Hermitian operator correspond to the possible measurement outcomes of an observable, and physical observables must have real values.

5. How are Hermitian operators used in quantum mechanics calculations?

Hermitian operators are used to represent physical observables in quantum mechanics, such as position, momentum, and energy. They are also used in solving the Schrödinger equation and calculating expectation values for measurements. Additionally, Hermitian operators can be used to construct other important operators, such as the Hamiltonian, which is used to describe the total energy of a system.

Similar threads

Replies
21
Views
2K
  • Quantum Physics
Replies
15
Views
2K
  • Quantum Physics
Replies
2
Views
1K
  • Quantum Physics
Replies
6
Views
1K
Replies
3
Views
887
Replies
1
Views
599
Replies
1
Views
2K
  • Quantum Physics
Replies
7
Views
1K
  • Quantum Physics
Replies
6
Views
817
Back
Top